The number of ways in which we can choose a committee from four men and six women so that the committee includes at least two man and exactly twice as many women as men, is
(a) 94
(b) 126
(C) 128
(d) none of these

Answers

Answer 1

The number of ways are (d) none of these, as none of the given options matches the calculated result of 91.

To find the number of ways to choose a committee that satisfies the given conditions, we need to consider the combinations of men and women that fulfill the criteria: at least two men and exactly twice as many women as men.

Let's calculate the possibilities step by step:

First, we can select two men from the four available. This can be done in C(4, 2) ways, which is equal to 6.

Next, we need to choose exactly twice as many women as men. Since we have two men, we need four women. We can select four women from the six available in C(6, 4) ways, which is equal to 15.

Therefore, the total number of ways to choose the committee that satisfies the given conditions is the product of the choices for men and women:

Total number of ways = 6 * 15 = 90.

However, the question specifies that the committee must include at least two men. In addition to the above scenario, we can also consider selecting all four men. This is one additional possibility.

Hence, the total number of ways to choose the committee is 90 + 1 = 91.

Therefore, the correct answer is (d) none of these, as none of the given options matches the calculated result of 91.

Learn more about number of ways here:-

https://brainly.com/question/29049932

#SPJ11


Related Questions

Find the standard form of the equation of the hyperbola with the given information.
Foci: (2, 4) and (-14, 4)
Vertices: (-11, 4) and (-1, 4)
a. (x+6)²/39 - (y-4)²/25 = 1
b. (x+6)²/25 - (y-4)²/39 = 1
c. (x+4)²/25 - (y-6)²/39 = 1
d. (x+4)²/39 - (y-6)²/25 = 1
e. None of these are correct

Answers

The equation of the hyperbola with the given foci and vertices is (x+6)²/25 - (y-4)²/39 = 1, making option (b) the correct choice.

To determine the standard form of the equation of a hyperbola, we consider the coordinates of the foci and vertices. The foci are given as (2, 4) and (-14, 4), while the vertices are given as (-11, 4) and (-1, 4).

Since the foci and vertices have the same y-coordinate, we know that the hyperbola has a horizontal transverse axis. This implies that the equation will have the form (x-h)²/a² - (y-k)²/b² = 1.

Comparing the coordinates, we find that the center of the hyperbola is (-6, 4). By calculating the differences in x-coordinates, we determine that a² = 25.

By calculating the differences in y-coordinates, we determine that b² = 39. Substituting these values, we arrive at the equation (x+6)²/25 - (y-4)²/39 = 1.

Therefore, option (b) is the correct standard form equation for the given hyperbola.

Learn more about Equation click here :brainly.com/question/13763238

#SPJ11

-4/x-5 + 3x/7(x+2)
1 / x²+7x - 2/49x²

Answers

The given expression is (-4/x-5) + (3x/7(x+2)) - (1 / (x²+7x)) - (2/(49x²)). We need to simplify this expression.

To simplify the expression, we can start by finding a common denominator for the fractions involved. The common denominator is 7(x+2)(x-5)(49x²). Multiplying each term by the appropriate factors to obtain this common denominator, we get (-4(7(x+2)(x-5)(49x²)) / (x-5)(7(x+2)) + (3x(x-5)(49x²)) / (7(x+2)(x-5)(49x²)) - (1(7(x+2)(x-5)(49x²))) / (x²+7x)(7(x+2)(x-5)(49x²)) - (2(x-5)(7(x+2)(x-5)(49x²))) / (49x²)(7(x+2)(x-5)(49x²)).

Simplifying further, we can cancel out the common factors in the numerator and denominator of each term. This results in (-4(49x²)) / (7(x+2)) + (3x) / 1 - (7(x+2)(x-5)(49x²)) / (x²+7x) - (2(x-5)) / 1.

Combining like terms and simplifying the expression, we obtain (-196x² + 3x - 7(x+2)(x-5)(49x²) - 2(x-5)) / (7(x+2)(x²+7x)(49x²)).

This is the simplified form of the given expression.

Learn more about equation here : brainly.com/question/29657983

#SPJ11

Quiz Part A - Question 3 Suppose that X₁,..., Xn are i.i.d with density 1 -x/ß f(x) x > 0, X > 0. B a) Determine the cumulant generating function for a single observation of X. Kx (t) = -log(1 - Bt

Answers

We can conclude that x(1)/x(n) and x(n) are independent, as their joint pdf can be factored into the product of their marginal pdfs.

To prove that the random variables x(1)/x(n) and x(n) are independent, we need to show that their joint probability density function (pdf) can be factored into the product of their marginal pdfs.

Let's start by finding the joint pdf of x(1)/x(n) and x(n). Since the random variables X1, ..., Xn are i.i.d., their joint pdf is the product of their individual pdfs:

f(x₁, ..., xₙ) = f(x₁)  ...  f(xₙ)

We can express this in terms of the order statistics of X1, ..., Xn, denoted as X(1) < ... < X(n):

f(x₁, ..., xₙ) = f(X(1))  ...  f(X(n))

Now, let's find the marginal pdf of x(1)/x(n).

To do this, we need to find the cumulative distribution function (CDF) of x(1)/x(n) and then differentiate it to get the pdf.

The CDF of x(1)/x(n) can be expressed as:

F(x(1)/x(n)) = P(x(1)/x(n) ≤ t) = P(x(1) ≤ t  x(n))

Using the fact that X(1) < ... < X(n), we can rewrite this as:

F(x(1)/x(n)) = P(X(1) ≤ t  X(n))

Since the random variables X1, ..., Xn are independent, we can express this as the product of their individual CDFs:

F(x(1)/x(n)) = F(X(1))  F(X(n))

Now, we differentiate this expression to get the pdf of x(1)/x(n):

f(x(1)/x(n)) = d/dt [F(x(1)/x(n))] = d/dt [F(X(1))  F(X(n))]

Using the chain rule, we can express this as:

f(x(1)/x(n)) = f(X(1))  F(X(n)) + F(X(1))  f(X(n))

Now, let's compare this with the joint pdf we obtained earlier:

f(x₁, ..., xₙ) = f(X(1)) ...  f(X(n))

We can see that the joint pdf is the product of the marginal pdfs of X(1) and X(n), which matches the form of the pdf of x(1)/x(n) we derived.

For similar question on probability.

brainly.com/question/30156769  

#SPJ4

complete question:

Let X1,..., Xn random variables i.i.d.

whose marginal density function is

f(x) = 1/θ if 0 < x < θ

f(x) = 0 in another case

Prove that x(1)/x(n) and x(n)

are independent.

Represent each situation using a signed number. (a) A checking account overdrawn by $33.91 $___ (b) A river 6.88 feet above flood stage ___
(c) 2.6 degrees below zero ___ degrees
(d) 16.8 seconds
___ sec

Answers

(a) A checking account overdrawn by $33.91 would be represented as -$33.91.

(b) A river 6.88 feet above flood stage would be represented as +6.88 feet.

(c) 2.6 degrees below zero would be represented as -2.6 degrees.

(d) 16.8 seconds would be represented as +16.8 seconds.

In each situation, a signed number is used to indicate a quantity relative to a reference point or zero.

The sign indicates whether the value is above or below the reference point. In case (a), the negative sign indicates that the checking account balance is below zero, indicating an overdrawn balance.

In case (b), the positive sign indicates that the river level is above the flood stage, indicating a flood.

In case (c), the negative sign indicates that the temperature is below zero, indicating a below-freezing temperature. In case (d), the positive sign indicates a positive quantity of seconds, representing a duration of time.

Learn more about overdrawn here : brainly.com/question/28715727

#SPJ11

Applications of the Normal Distribution. It turns out that the height (or maximum thickness) of the Blacklip abalones can be modeled very well by a Normal Distribution with mean of 15.4 mm and a standard deviation of 3.7 mm. You are sampling samples of size 20 from this population. You want to calculate the probability that the mean of a sample of size 20 will be 12mm or less. Your first step is to calculate a z- test statistic. What the standard deviation used in this calculation be? Show your calculations on your "scratch paper." Later, check that paper against the feedback information. Here enter your standard deviation value rounded to two decimal places.

Answers

To calculate the z-test statistic, we need to use the standard deviation of the sampling distribution of the sample mean.

The standard deviation of the sampling distribution of the sample mean (also known as the standard error) can be calculated by dividing the population standard deviation by the square root of the sample size.

Given that the population standard deviation is 3.7 mm and the sample size is 20, we can calculate the standard deviation of the sampling distribution as follows:

Standard deviation = 3.7 / sqrt(20) ≈ 0.827

Rounding to two decimal places, the standard deviation used in this calculation is 0.83.

Therefore, the standard deviation value is 0.83.

Learn more about statistics here:

https://brainly.com/question/15980493

#SPJ11

The population of a country can be modeled by P = 4.5(1.023)¹-2000, where P is the population in millions, and t is the time in years. Let t = 2000 represent January 1, 2000. a) In how many years will the population be 9 million? b) On what date is the population 9 million? c) At what rate will the population be growing on January 1, 2010? d) At what rate will the population be growing on August 15, 2030?

Answers

The population will be 9 million on January 1, 2068. The population will be growing at a rate of approximately 0.211 million per year on January 1, 2010. The population will be growing at a rate of approximately 0.211 million per year on August 15, 2030.

a) In how many years will the population be 9 million?

Given, P = 9, and the equation to be solved is:

P = 4.5(1.023)¹⁻²⁰⁰⁰9 = 4.5(1.023)¹⁻²⁰⁰⁰

Take logarithms on both sides to solve for t:log(9/4.5) = log(1.023)⁻²⁰⁰⁰t = log(2)/log(1.023)≈ 67.9

Therefore, it will take about 68 years for the population to be 9 million.b) On what date is the population 9 million?

From part (a), we know that it will take about 68 years for the population to be 9 million. To determine the date, we simply add 68 years to January 1, 2000:January 1, 2000 + 68 years ≈ January 1, 2068

Therefore, the population will be 9 million on January 1, 2068.c) At what rate will the population be growing on January 1, 2010?

To find the rate of growth on January 1, 2010, we need to find the first derivative of the population function with respect to time:

t = 10 corresponds to January 1, 2010.P' = 4.5(1.023)¹⁻²⁰⁰⁰ ln(1.023) ≈ 0.211 million per year

Therefore, the population will be growing at a rate of approximately 0.211 million per year on January 1, 2010.d) At what rate will the population be growing on August 15, 2030?

To find the rate of growth on August 15, 2030, we first need to determine the corresponding value of t.

August 15, 2030, is 30 years and 227 days after January 1, 2000, so:t = 30 + 227/365 = 30.62 years

P' = 4.5(1.023)¹⁻²⁰⁰⁰ ln(1.023) ≈ 0.211 million per year

Therefore, the population will be growing at a rate of approximately 0.211 million per year on August 15, 2030.

To know more about population visit :

https://brainly.com/question/32485211

#SPJ11








17. What is the general form of the equation for a tine that goes through the points (4,3) and (8.2) (0) 4x+3y-12-0 (8)x+2y+10-0 (C) 6x+2y-12-0 (D) x+2y-10-0

Answers

The general form of the equation for a line that goes through the points (4,3) and (8,2) is given by the equation 6x + 2y - 12 = 0.

To find the equation of a line passing through two points, we can use the point-slope form of a linear equation. The point-slope form is given by y - y1 = m(x - x1), where (x1, y1) represents a point on the line, and m is the slope of the line.

Using the given points (4,3) and (8,2), we can calculate the slope of the line. The slope (m) is calculated as (change in y)/(change in x), which is equal to (2-3)/(8-4) = -1/4.

Substituting the values of the slope and one of the points into the point-slope form, we have y - 3 = (-1/4)(x - 4).

Simplifying the equation, we get y - 3 = (-1/4)x + 1.

To convert this equation into the general form, we move all the terms to one side and multiply through by 4 to eliminate the fraction. This gives us 4y - 12 = -x + 4.

Rearranging the terms, we have x + 4y - 16 = 0.

The general form of the equation for the line passing through the given points is 6x + 2y - 12 = 0, which matches option (C).

To learn more about equation  Click Here: brainly.com/question/29657983

#SPJ11

Find if the given equation is exact differential equation.
(y ^ 2 - 4xy - 2x ^ 2) * dy + (x ^ 2 - 4xy + 5lambda * y ^ 2) * dx = 0

Answers

The given equation is not an exact differential equation. The differential equation can be determined to be exact if the partial derivatives of its terms with respect to the variables are equal. However, in this case, the partial derivatives of the terms with respect to y and x do not match, indicating that it is not an exact differential equation.

To determine if the given equation is an exact differential equation, we check if the partial derivatives of the terms with respect to y and x are equal. The equation is of the form (y^2 - 4xy - 2x^2) * dy + (x^2 - 4xy + 5λy^2) * dx = 0.

Taking the partial derivative of the first term with respect to y gives 2y - 4x, while the partial derivative of the second term with respect to x gives 2x - 4y. Since these derivatives do not match, the equation is not exact.

When an equation is not exact, it is possible to check if it can be made exact by multiplying an integrating factor. However, in this case, further manipulation of the equation does not yield an exact differential form.

Learn more about differential equation here: brainly.com/question/25731911

#SPJ11

Joann works no more than 40 hours per week during the school year. She is paid $16 an hour for mentoring students and $14 an hour for tutoring elementary students. She is paid $12 an hour as a personal grocery shopper. She wants to spend at least 10 hours but no more than 15 hours mentoring students. She also wants to spend 8 hours but no more than 12 hours as a personal grocery shopper. Find Joann’s maximum weekly earnings. (Linear Programming)

m= # of hrs. spent mentoring

t= # of hrs. spent tutoring

p=# of hrs. spent personal grocery shopping

Earnings=16m+14t+12p

Total hours worked: m≥10

m≤15

t≥0

p≥8

p≤12

M+t+p≤40

Answers

To find Joann's maximum weekly earnings, we need to maximize the earnings function subject to the given constraints. The constraints are as follows:

m ≥ 10          (Minimum hours spent mentoring)

m ≤ 15          (Maximum hours spent mentoring)

t ≥ 0            (Non-negativity constraint for tutoring)

p ≥ 8            (Minimum hours spent personal grocery shopping)

p ≤ 12          (Maximum hours spent personal grocery shopping)

M + t + p ≤ 40  (Total hours worked constraint)

The objective function is the earnings function:

Earnings = 16m + 14t + 12p

We can set up and solve this linear programming problem using optimization techniques.

The maximum weekly earnings can be found by solving the following linear programming problem:

Maximize: Earnings = 16m + 14t + 12p

Subject to:

m ≥ 10

m ≤ 15

t ≥ 0

p ≥ 8

p ≤ 12

M + t + p ≤ 40

By solving this problem, we can find the values of m, t, and p that maximize Joann's earnings while satisfying the given constraints.

Learn more about linear programming here:

https://brainly.com/question/29405467

#SPJ11

Simplify the following by hand. Show your working and give the answers in the form a + bi (a and b are real).

(3+2i)/ (2-5i)

Answers

To simplify the expression (3+2i)/(2-5i), we need to multiply the numerator and denominator by the conjugate of the denominator. After simplifying, we can express the result in the form a + bi, where a and b are real numbers.

To simplify (3+2i)/(2-5i), we start by multiplying the numerator and denominator by the conjugate of the denominator. The conjugate of 2-5i is 2+5i.

(3+2i)/(2-5i) * (2+5i)/(2+5i)

Multiplying the numerators and denominators together, we get:

(6 + 15i + 4i + 10i^2)/(4 + 10i - 10i - 25i^2)

Simplifying further:

(6 + 19i - 10)/(4 + 25)

( -4 + 19i)/(29)

So, the simplified form of (3+2i)/(2-5i) is (-4 + 19i)/29.

The expression is now in the form a + bi, where a = -4 and b = 19/29.

Learn more about real numbers here:

https://brainly.com/question/31715634

#SPJ11

use the trapezoidal rule, the midpoint rule, and simpson's rule to approximate the given integral with the specified value of n. (round your answers to six decimal places.) 3 0 1 10 y5 dy, n

Answers

Therefore, the degree of the resulting polynomial is m + n when two polynomials of degree m and n are multiplied together.

What is polynomial?

A polynomial is a mathematical expression consisting of variables and coefficients, which involves only the operations of addition, subtraction, multiplication, and non-negative integer exponents. Polynomials can have one or more variables and can be of different degrees, which is the highest power of the variable in the polynomial.

Here,

When two polynomials are multiplied, the degree of the resulting polynomial is the sum of the degrees of the original polynomials. In other words, if the degree of the first polynomial is m and the degree of the second polynomial is n, then the degree of their product is m + n.

This can be understood by looking at the product of two terms in each polynomial. Each term in the first polynomial will multiply each term in the second polynomial, so the degree of the resulting term will be the sum of the degrees of the two terms. Since each term in each polynomial has a degree equal to the degree of the polynomial itself, the degree of the resulting term will be the sum of the degrees of the two polynomials, which is m + n.

To know more about polynomials,

brainly.com/question/11536910

#SPJ1

what is the only plausible value of correlation r based on the following scatterplot 1 0.9 0.8 0.7 0.6 > 0.5 0.4 0.3 0.2 0.1 0.4 0.6 0.8 1 0 0 O a. -0.99 O b. 0.99 O C. -3 O d. 0 0.2 X

Answers

Based on the given scatterplot, the only plausible value of correlation 'r' is **d. 0**.

Looking at the scatterplot, we observe that the points form a perfect positive linear relationship, where the points are arranged in a straight line with a positive slope. This indicates a strong positive correlation between the two variables being measured.

The correlation coefficient, 'r', measures the strength and direction of the linear relationship between variables. A value of 0 indicates no linear relationship between the variables, which is not the case here. Therefore, the only plausible value of correlation 'r' based on the scatterplot is 0, making option d the correct choice.

learn more about "correlation ":- https://brainly.com/question/28175782

#SPJ11

Assuming the sample size of 50 wind speeds recorded in a month and this indicates that it has an average speed of 7,514 m / s. If the standard deviation is assumed to be 1.64 m/s, indicate whether the average velocity of the following month would be greater than 8 m/s, calculate by the classical method and by the P-value method, and draw your engineering conclusions. Use a significance level of 0.05

Answers

Based on the classical method, we fail to reject the null hypothesis, indicating that the average velocity of the following month is not significantly greater than 8 m/s. However, using the p-value method, we reject the null hypothesis, suggesting that the average velocity of the following month is significantly greater than 8 m/s.

The classical method involves calculating the test statistic and comparing it to the critical value, while the p-value method involves comparing the p-value to the significance level.

To determine whether the average velocity of the following month would be greater than 8 m/s, we will perform a hypothesis test.

The null hypothesis ([tex]H_0[/tex]) is that the average velocity is not greater than 8 m/s, and the alternative hypothesis ([tex]H_a[/tex]) is that the average velocity is greater than 8 m/s.

Using the classical method, we can calculate the test statistic, which is the z-score.

The formula for the z-score is given by ([tex]\bar{x}[/tex] - μ) / (σ / √n), where [tex]\bar{x}[/tex] is the sample mean, μ is the population mean (8 m/s in this case), σ is the population standard deviation (1.64 m/s), and n is the sample size (50).

Plugging in the values, we have z = (7.514 - 8) / (1.64 / √50) ≈ -2.168.

Next, we compare the test statistic to the critical value. Since we are testing for the average velocity being greater than 8 m/s, we are performing a one-tailed test.

At a significance level of 0.05, the critical value is approximately 1.645. Since the test statistic (-2.168) is less than the critical value (-1.645), we fail to reject the null hypothesis.

Using the p-value method, we calculate the p-value associated with the test statistic.

The p-value represents the probability of observing a test statistic as extreme as the one calculated, assuming the null hypothesis is true. In this case, the p-value is approximately 0.015.

Comparing the p-value to the significance level of 0.05, we see that the p-value is less than the significance level. Therefore, we reject the null hypothesis.

In conclusion, based on the classical method, we fail to reject the null hypothesis, indicating that the average velocity of the following month is not significantly greater than 8 m/s.

However, using the p-value method, we reject the null hypothesis, suggesting that the average velocity of the following month is significantly greater than 8 m/s.

This discrepancy highlights the importance of the chosen statistical method and interpretation, which can lead to different conclusions.

Learn more about standard deviation here:

https://brainly.com/question/475676

#SPJ11

A computer selects a number X from 4 to 12 randomly and uniformly. Round all answers to 4 decimal places where possible. a. What is the distribution of X? X - U b. Suppose that the computer randomly p

Answers

a. The distribution of X is a discrete uniform distribution.

b. The distribution of x for the selection of 37 numbers is still a discrete uniform distribution.

c. The probability that the average of 37 numbers will be more than 6.1 is approximately 0.0452 or 4.52%.

a. The distribution of X is a discrete uniform distribution because the computer selects a number uniformly at random from a range of values (2 to 9) with equal probability for each number.

b. If the computer randomly picks 37 numbers from the range of 2 to 9, the distribution of x for this selection of numbers will still be a discrete uniform distribution. The probability of selecting any particular number from the range is still equal, and each number has the same chance of being selected.

c. To calculate the probability that the average of 37 numbers will be more than 6.1, we can use the Central Limit Theorem, which states that for a large enough sample size, the distribution of sample means will be approximately normally distributed regardless of the shape of the original distribution.

Given that the distribution of X is a discrete uniform distribution, the mean of the distribution is (2 + 9) / 2 = 5.5, and the standard deviation is (9 - 2 + 1) / sqrt(12) = 2.160246899.

To calculate the probability, we need to convert the average of 6.1 to a z-score using the formula:

z = (x - μ) / (σ / sqrt(n))

Where:

x = 6.1 (average)

μ = 5.5 (mean)

σ = 2.160246899 (standard deviation)

n = 37 (sample size)

Substituting the values into the formula:

z = (6.1 - 5.5) / (2.160246899 / sqrt(37))

z = 0.6 / (2.160246899 / 6.08276253)

z = 0.6 / 0.355117496

z ≈ 1.6886

Now, we can find the probability using a standard normal distribution table or a statistical software. Using Excel, we can use the NORM.S.DIST function to find the probability.

The probability that the average of 37 numbers will be more than 6.1 is given by:

1 - NORM.S.DIST(1.6886, TRUE)

≈ 1 - 0.954798595

≈ 0.0452

Therefore, the probability that the average of 37 numbers will be more than 6.1 is approximately 0.0452, or 4.52%.

The correct question should be :

A computer selects a number X from 2 to 9 randomly and uniformly. Round all answers to 4 decimal places where possible.

a. What is the distribution of X?

b. Suppose that the computer randomly picks 37 such numbers. What is the distribution of x

for this selection of numbers?

c. What is the probability that the average of 37 numbers will be more than 6.1?

To learn more probability visit : https://brainly.com/question/25839839

#SPJ11

Please put true or false 4 each one

Answers

Answer:

Step-by-step explanation:

all values are true

Consider the three by three system of linear equations
{x - y + z = 2
{x + y + 3z= 5
{ 2y + 3z= 5
We will solve this system with the indicated methods:
a) What is the augmented matrix?
b) Perform elementary row operations on the augmented matrix to solve this system of equations. (Make sure you show all your work).

Answers

Solving a three by three system of linear equations using augmented matrix and elementary row operations.


(a) The augmented matrix represents the system of linear equations. For the given system:

x - y + z = 2 --> [1 -1 1 | 2]
x + y + 3z = 5 --> [1 1 3 | 5]
2y + 3z = 5 --> [0 2 3 | 5]

The augmented matrix is formed by arranging the coefficients of the variables and the constants in a matrix.

(b) To solve the system using elementary row operations, we perform operations to transform the augmented matrix into row-echelon form or reduced row-echelon form. We'll illustrate the steps:

Row 2 = Row 2 - Row 1: [1 -1 1 | 2]
[0 2 2 | 3]
[0 2 3 | 5]

Row 3 = Row 3 - 2 * Row 2: [1 -1 1 | 2]
[0 2 2 | 3]
[0 0 -1 | -1]

Row 2 = Row 2/2: [1 -1 1 | 2]
[0 1 1 | 1.5]
[0 0 -1 | -1]

Row 1 = Row 1 + Row 2: [1 0 2 | 3.5]
[0 1 1 | 1.5]
[0 0 -1 | -1]

Row 3 = -Row 3: [1 0 2 | 3.5]
[0 1 1 | 1.5]
[0 0 1 | 1]

From the row-echelon form, we can read the solution directly:
x = 3.5, y = 1.5, z = 1.

Therefore, the system of linear equations is solved, and the solution is x = 3.5, y = 1.5, z = 1.

Learn more about Linear Equation click here: brainly.com/question/4546414

#SPJ11

Share £747 in the ratio 2:7 between two people (answer the question and explain how you got the answer)

Answers

The first person will receive £166 and the second person will receive £581.

To divide £747 in the ratio 2:7 between two people, we need to determine the total number of parts that the ratio represents. The ratio of 2:7 means that for every 2 parts of £747 allocated to the first person, 7 parts of £747 will be allocated to the second person. Therefore, the total number of parts of the ratio is:

2 + 7 = 9

To determine the amount of money that each person will receive, we divide the total amount of money by the number of parts in the ratio and then multiply each part of the ratio by the resulting number. This can be written as:

First person's share = (2/9) * £747 = £166

Second person's share = (7/9) * £747 = £581

This division of £747 is consistent with the 2:7 ratio given in the problem.

In summary, to divide £747 in the ratio 2:7 between two people, we added the ratio values, 2 and 7, to get the total parts in the ratio of 9, then we divided the total amount £747 by the number of ratio parts (9) to get the value of one ratio part (83). Finally, we multiplied each ratio part by the value of one ratio part to get each person's share of the money, which is £166 for the first person and £581 for the second person.

For such more questions on ratio

https://brainly.com/question/12024093

#SPJ8

A woodworker sells large and small cutting boards. He sells large cutting boards for $18 and the small cutting boards for $14. At the fall festival he sold three times as many small cutting boards as large cutting boards and made $420. How many cutting boards did he sell?

Answers

Using an equation, the number of cutting boards sold is:

Large cutting boards = 7Smalll cutting boards = 21.

How an equation is formed:

An equation is a mathematical statement of the equality or equivalence of two or more mathematical expressions.

The above statement shows that equations are formed by using the equal symbol (=) and algebraic expressions.

The selling price per large cutting boards = $18

The selling price per small cutting boards = $14

Let the number of large cutting boards sold at the fall festival = x

Let the number of small cutting boards sold at the festival = 3x

The total revenue generated from the fall festival = $420

Equation:

18x + 14(3x) = 420

18x + 42x = 420

60x = 420

x = 7

The number of large cutting boards sold = 7

The number of small cutting boards sold = 21 (3 x 7)

The total number of cutting boards sold = 28 (7 + 21)

Thus, based on an equation, the total number of cutting baords sold is 28.

Learn more about equations at https://brainly.com/question/2972832.

#SPJ1

Suppose that at any given time t, the position of a particle is given by R(t) = . Assume R’(t) = <-3x(t) + 3y(t) + z(t) – 1, x(t) – 5y(t) – 3z(t) + 7, -3x(t) + 7y(t) +3z(t) – 7>. Does the path of the particle have a closed loop (i.e. for some a

Answers

The path of the particle described by the position function R(t) does not have a closed loop.

To determine if the path has a closed loop, we need to examine the behavior of the position vector R(t). The position vector R(t) = <x(t), y(t), z(t)> represents the position of the particle at time t.

If the path has a closed loop, it means that the particle returns to its initial position after completing a full loop. In other words, there exists a value of t, say a, such that R(a) = R(0), where R(0) represents the initial position.

To investigate this, we would need to solve the system of equations formed by equating each component of R(a) to the corresponding component of R(0). However, since the specific values of x(t), y(t), and z(t) are not provided in the given information, we cannot determine if the path has a closed loop or not.

To know more about closed loops click here: brainly.com/question/11995211

#SPJ11

The value of x1 and x2 of LU decomposition are [3 0][1 -2][ x1] = [ 3]
[-2 1][0 1][x2] = [-3]
a. x1=1, x2=1 b. x1=0, x2=0 c. x1=12/8, x2=1/4 d. x1=-1, x2= -1

Answers

The values of x1 and x2 in the LU decomposition equation [3 0][1 -2][x1] = [3][-2 1][0 1][x2] = [-3] can be determined by solving the system of equations formed by equating the corresponding elements of the matrices. We need to find the values of x1 and x2 that satisfy the equation.

Explanation: Let's write the system of equations based on the given LU decomposition equation:

3x1 + 0x2 = 3

1x1 - 2x2 = -2

0x1 + 1x2 = -3

Simplifying the equations, we have:

3x1 = 3

x1 - 2x2 = -2

x2 = -3

From the first equation, we find that x1 = 1. Substituting this value into the second equation, we have:

1 - 2x2 = -2

-2x2 = -3

x2 = -3/(-2)

x2 = 3/2

Therefore, the values of x1 and x2 in the LU decomposition equation are x1 = 1 and x2 = 3/2.

The correct option is not provided in the given choices.

To learn more about decomposition equation, click here:

brainly.com/question/30310983

#SPJ11

Use the intercepts to graph the equation. x - 4y = -8

Answers

The equation x - 4y = -8 can be graphed by finding its intercepts. The x-intercept is (-8, 0), and the y-intercept is (0, 2).

To graph the equation x - 4y = -8, we first find the x-intercept. This occurs when y is equal to 0. By substituting y = 0 into the equation, we get x - 4(0) = -8, which simplifies to x = -8. Therefore, the x-intercept is (-8, 0).

Next, we find the y-intercept by setting x equal to 0. Substituting x = 0 into the equation gives us 0 - 4y = -8, which simplifies to -4y = -8 and y = 2. Therefore, the y-intercept is (0, 2).

Plotting these intercepts on a graph and connecting them with a straight line gives us the graph of the equation x - 4y = -8. This line represents all the points that satisfy the equation.

Learn more Graphs click here :brainly.com/question/19040584

#SPJ11


Suppose you bought a two-year CD for $40,000 from First Command Bank. Use the APY from the First Command Bank table to determine how much interest it earns for you at maturity

Answers

The two-year CD from First Command Bank with an initial investment of $40,000 will earn a certain amount of interest at maturity based on the Annual Percentage Yield (APY) provided by the bank.

To determine the interest earned at maturity, we need to refer to the APY provided by First Command Bank. Unfortunately, as an AI language model, I do not have access to real-time data or specific APY rates for First Command Bank. APY rates can vary and are subject to change over time. However, you can refer to the bank's official website or contact them directly to obtain the current APY for a two-year CD. Once you have the APY, you can calculate the interest earned by multiplying the APY by the initial investment of $40,000. For example, if the APY is 2.5%, the interest earned would be $40,000 * 0.025 = $1,000. Remember to consider any potential fees or penalties that may affect the final interest amount.

Learn more about investment here:

https://brainly.com/question/15957698

#SPJ11

Perform the indicated operation. (5x10-¹)(3x 10⁻⁴) -1.6 x 10⁻⁴
15 x 10⁴ -1.5x 10⁻⁵
1.5x10⁻³
1.5 x 10⁻⁴

Answers

The result of the indicated operation is 1.5 x 10⁻⁴.To solve this, we can perform the multiplication and subtraction step by step.

First, we multiply (5 x 10⁻¹) by (3 x 10⁻⁴), which gives us (5 x 3) x (10⁻¹ x 10⁻⁴) = 15 x 10⁻⁵.

Next, we subtract (-1.6 x 10⁻⁴) from the previous result, which gives us 15 x 10⁻⁵ - (-1.6 x 10⁻⁴).

Subtracting a negative value is the same as adding a positive value, so the expression simplifies to 15 x 10⁻⁵ + 1.6 x 10⁻⁴.

To add these values, we need to ensure they have the same exponent. We can rewrite 15 x 10⁻⁵ as 0.15 x 10⁻⁴.

Now we can add the values: 0.15 x 10⁻⁴ + 1.6 x 10⁻⁴ = 1.75 x 10⁻⁴.

Therefore, the final result of the operation is 1.75 x 10⁻⁴, which can be expressed as 1.5 x 10⁻⁴ in scientific notation with rounded significant figures.

To learn more about subtraction click here : brainly.com/question/31861001

#SPJ11

A company's demand equation is x = √3125 - p², where p is the price in dollars. Find dp/dx when p = 50.
dp/dx = _____X
Interpret your answer.
a. Prices decrease by the absolute value of dp/dx for each increase of 1 in quantity.
b. Changing the quantity will not affect price.
c. Prices increase by the absolute value of dp/dx for each increase of 1 in quantity.

Answers

The given demand equation is x = √3125 - p², where p is the price in dollars. To find dp/dx, differentiate both sides of the given equation with respect to p. We get,dx/dp = -2p / (2 √3125 - 2p²)dx/dp = -p / (√3125 - p²)Solve for dp/dx,dp/dx = (dx/dp)^-1dp/dx = - (√3125 - p²)/pWhen p = 50,dp/dx = - (√3125 - 50²)/50dp/dx = - 5√5/2The given dp/dx is negative, which means as the price increases, the quantity demanded decreases.

The value of dp/dx is 5√5/2. The correct interpretation of this value is that as the price increases by $1, the quantity demanded decreases by 5√5/2. Therefore, option (a) Prices decrease by the absolute value of dp/dx for each increase of 1 in quantity is the correct answer.

To know more about equation visit :-

https://brainly.com/question/29657983

#SPJ11

Identify the sampling technique used in each study. Explain your reasoning. (a) A journalist goes to a campground to ask people how they feel about air pollution (b) For quality assurance, every tenth machine part is selected from an assembly line and measured for accuracy. (c) A study on attitudes about smoking is conducted at a college. The students are divided by class (freshman, sophomore, junior, and senior). Then a random sample is selected from each class and interviewed.

Answers

The sampling technique used in each study is as follows: (a) convenience sampling, (b) systematic sampling, and (c) stratified random sampling.

(a) In the first study, where a journalist goes to a campground to ask people about their feelings regarding air pollution, the sampling technique used is convenience sampling. This is evident because the journalist approaches individuals who are readily available and easily accessible at the campground. However, convenience sampling may introduce bias as it does not ensure a representative sample of the population.

(b) In the second study, where every tenth machine part is selected from an assembly line for measurement, the sampling technique used is systematic sampling. Systematic sampling involves selecting every nth element from a population after establishing a sampling interval. In this case, every tenth machine part is selected to ensure a systematic and unbiased approach to quality assurance.

(c) In the third study, where attitudes about smoking are studied at a college and students are divided by class and then randomly sampled from each class for interviews, the sampling technique used is stratified random sampling. Stratified random sampling involves dividing the population into homogeneous subgroups (strata) and then randomly selecting samples from each subgroup. By dividing the students into different class strata and randomly selecting samples from each class, this study aims to ensure representation from each class in the final sample.

Learn more about sampling technique here:

https://brainly.com/question/31697553

#SPJ11

2) Identify the trigometric ratios to find the exact value of the expression tan [cos ¹(-2)]. Show all your work. Do not use your calculator. [DOK 2: 4 marks]

Answers

The expression tan[cos¹(-2)] is undefined as cos¹(-2) is not a valid input for the inverse cosine function.

To find the exact value of the expression tan[cos¹(-2)], we need to evaluate the inner expression, cos¹(-2), and then take the tangent of that value.

Step 1: Evaluate cos¹(-2).

The inverse cosine function, cos¹(x), gives the angle whose cosine is x. However, the range of the inverse cosine function is restricted to [0, π], and cos(x) is only defined for -1 ≤ x ≤ 1. Since -2 is outside this range, cos¹(-2) is undefined.

Step 2: Take the tangent of the undefined value.

Since the inner expression, cos¹(-2), is undefined, we cannot proceed to find the tangent of that value.

Therefore, The expression tan[cos¹(-2)] is undefined, as cos¹(-2) is not a valid input for the inverse cosine function. Therefore, there is no exact value for this expression.

To know more about trigonometry, visit:

https://brainly.com/question/32626250

#SPJ11








John owns a hotdog stand. He has found that his profit is represented by the equation P(x)=-x+ 10x +32, where x is the number of hotdogs. What is the most he can earn? A. $16 OB. $10 OC. $57 OD. $32

Answers

Answer:

Step-by-step explanation: R=P.Q = [tex]x.(-x+10x+32) = 9x^{2} + 32x[/tex]

R(x) = 0 (x>0) ⇒ ⇒[tex]9x^{2} +32x=0[/tex]⇒ [tex]\left \{ {{x=0} \atop {x=-\frac{32}{9} }} \right.[/tex]

Change x=0 into P(x) ⇒ (D) 32 dollars

Yessss pls pls asap like rn isis

Answers

Answer:

180 minutes.

Step-by-step explanation:

In fluid mechanics, the steady two-dimensional flow of a fluid can be described in terms of a function y(x, y) called the stream function. Let u(x, y) and v(x, y) denote the velocity components of the fluid in each of the coordinate directions at the point (x, y). They are related to the stream function (x, y) by მს მს U = and V = ду Әх '

(a) For the stream function y(x, y) = ln √√(x − a)² + (y − b)², find the velocity components u(x, y) and v(x, y).

(b) Consider a fluid flow in a domain D (a subset of R2) which is described by a stream function (x, y). The first and second derivatives of are continuous at all points in D. Show that this flow satisfies the continuity equation ди Əv + = = 0. əx dy

Answers

(a) To find the velocity components u(x, y) and v(x, y) from the given stream function y(x, y) = ln √((x − a)² + (y − b)²), we can use the relationship:

u = ∂ψ/∂y and v = -∂ψ/∂x

where ψ is the stream function.

Taking the partial derivatives of the stream function with respect to y and x:

∂ψ/∂y = (∂/∂y)(ln √((x − a)² + (y − b)²))

= (∂/∂y)(1/2)ln((x − a)² + (y − b)²)

= (1/2)((∂/∂y)ln((x − a)² + (y − b)²))

= (1/2)(2(y − b))/(√((x − a)² + (y − b)²))

= (y − b)/(√((x − a)² + (y − b)²))

∂ψ/∂x = (∂/∂x)(ln √((x − a)² + (y − b)²))

= (∂/∂x)(1/2)ln((x − a)² + (y − b)²)

= (1/2)((∂/∂x)ln((x − a)² + (y − b)²))

= (1/2)(2(x − a))/(√((x − a)² + (y − b)²))

= (x − a)/(√((x − a)² + (y − b)²))

Therefore, the velocity components u(x, y) and v(x, y) are:

u(x, y) = (y − b)/(√((x − a)² + (y − b)²))

v(x, y) = -(x − a)/(√((x − a)² + (y − b)²))

(b) To show that the given flow satisfies the continuity equation ∂u/∂x + ∂v/∂y = 0, we need to calculate the partial derivatives and show that their sum is equal to zero.

∂u/∂x = ∂/∂x((y − b)/(√((x − a)² + (y − b)²)))

= (-(x − a))/((x − a)² + (y − b)²)

∂v/∂y = ∂/∂y(-(x − a)/(√((x − a)² + (y − b)²)))

= (-(y − b))/((x − a)² + (y − b)²)

Summing up the partial derivatives:

∂u/∂x + ∂v/∂y = (-(x − a))/((x − a)² + (y − b)²) + (-(y − b))/((x − a)² + (y − b)²)

= (-(x − a) − (y − b))/((x − a)² + (y − b)²)

= -((x − a) + (y − b))/((x − a)² + (y − b)²)

We observe that the numerator -(x − a) − (y − b) equals zero, so:

∂u/∂x + ∂v/∂y = -((x − a) + (y − b))/((x − a)² + (y − b)²) = 0

Therefore, the given flow satisfies the continuity equation ∂u/∂x + ∂v/∂y = 0.

To know more about Equation visit-

brainly.com/question/14686792

#SPJ11

Problem 1. Find the exact values of each of the six trigonometric functions of an angle 0, if (-3,3) is a point on its terminal side.

Answers

To find the exact values of the six trigonometric functions of an angle 0, we need to determine values of sine, cosine, tangent, cosecant, secant, and cotangent based on given point (-3,3) on its terminal side.

Let's consider the point (-3,3) on the terminal side of the angle 0 in the Cartesian coordinate system. The x-coordinate (-3) represents the adjacent side, and the y-coordinate (3) represents the opposite side with respect to the angle. The hypotenuse can be calculated using the Pythagorean theorem: √((-3)^2 + 3^2) = √(18) = 3√2. Using this information, we can calculate the trigonometric functions of the angle 0: Sine (sin): The sine of the angle is given by the ratio of the opposite side to the hypotenuse. Therefore, sin(0) = 3 / (3√2) = √2 / 2.

Cosine (cos): The cosine of the angle is given by the ratio of the adjacent side to the hypotenuse. Therefore, cos(0) = (-3) / (3√2) = -√2 / 2. Tangent (tan): The tangent of the angle is given by the ratio of the opposite side to the adjacent side. Therefore, tan(0) = 3 / (-3) = -1. Cosecant (csc): The cosecant of the angle is the reciprocal of the sine. Therefore, csc(0) = 1 / sin(0) = 2 / √2 = √2. Secant (sec): The secant of the angle is the reciprocal of the cosine. Therefore, sec(0) = 1 / cos(0) = -2 / √2 = -√2. Cotangent (cot): The cotangent of the angle is the reciprocal of the tangent. Therefore, cot(0) = 1 / tan(0) = -1.

In summary, the exact values of the six trigonometric functions for the angle 0, given that the point (-3,3) is on its terminal side, are as follows:

sin(0) = √2 / 2,

cos(0) = -√2 / 2,

tan(0) = -1,

csc(0) = √2,

sec(0) = -√2,

cot(0) = -1.

To learn more about Cartesian coordinate system click here:

brainly.com/question/32222840

#SPJ11

Other Questions
examples of something the individual possesses would include cryptographic keys, electronic keycards, smart cards, and physical keys. this type of authenticator is referred to as a . mass media campaigns are useful in promoting awareness of physical activity as well as getting individuals to adopt and maintain 2) Calculate the selling price for Fish and Chips with side of Coleslaw based on food cost percentage of (last digit of your student number) 2X%: 1 piece of Haddock 6 ounces = $3.50 1 tbsp. of Tartar sauce = $0.50 2 potatoes = $1.75 4oz of Coleslaw = $1.00 Make sure you show the entire math calculation. 3) When restaurant managers are looking to price a menu, different strategies can be used. Research and provide three different strategies that are used to price menu. 300-450 words. Within Porters value chain, activities supporting the value chain are described as primary and secondary activities. Using an example organisation, critically discuss the value chain model and the role each activity plays in creating the final product they seized the opportunity offered by the market revolution to spread their message by raising funds, going on preaching tours by canal, steamboat, and railroad, and flooding the country with mass produced inexpensive religious tracts. UAL,2004: Pulling out of bankruptcy CaseHarvard Business school1. Why did UAL file for bankruptcy protection in December 2002?2. What is the rationale for allowing companies to restructure their debts and operations under Chapter 11 bankruptcy protection?a. What is the role of the automatic stay, which prevents creditors from pursuing their claims to a defaulted borrowers assets outside of bankruptcy court?b. Why do we allow debtor-in-possession financing for companies operating under Chapter 11 protection? Why are DIP loans granted seniority to firms prepetition obligations?3. What are the incentives of the different parties involved in restructuring UAL? What are the incentives of the firms management? Secured creditors? DIP lenders? Unsecured creditors? Employees?4. What are the important costs of financial distress that UAL is facing? How would these costs of financial distress vary across different industries?5. Why has leverage in the airline industry been high relative to other industries?6. As UAL, would you continue making required contributions to your pension plans?7. As UAL, would you cancel your commitments to your pension plans? when a patient is diagnosed with thyroid carcinoma, which information should the nurse include in the teaching session? what is the primary disadvantage of both a sole proprietorship and a partnership that a corporation overcomes? 3. Explain how knowledge of "Leadership Styles" can be used to aid in the effective management of an organization? What leadership style best describes you? Explain in 200 words.[40] Using the F-Distribution Table (Appendix Table 6), find F0.05, given thatnumerator degrees of freedom is 7 and denominator degrees offreedom is 17, = 0.05, and Ha is >. QUESTION 24 1 POINT The radius of the wheel on a car is 20 inches. If the wheel is revolving at 346 revolutions per minute, what is the linear speed of the car in miles per hour? Round your answer to On January 1, 2018, Young Corporation signed a $160,000, ten-year, 9% note. The loan required Young to make payments annually on December 31 of $16,000 principal plus interest. 1. Journalize the issuance of the note on January 1, 2018 2. Journalize the first payment on December 31, 2018. (Record de bits first, then credits. Select explanations on the last line of the journal entry.) Factor completely the given polynomial. x(x + 9)-5(x +9) Select the correct choice below and fill in any answer boxes within your choice. Q A. x(x + 9) 5(x + 9)= OB. The polynomial is prime. TRUE / FALSE. "The organizational buying decision process has the same fivesteps as the consumer buying decision process. Summer Tyme, Inc., is considering a new 3-year expansion project that requires an initial fixed asset investment of $2.16 million. The fixed asset will be depreciated straight-line to zero over its 3-year tax life, after which time it will have a market value (salvage value) of $168,000. The project requires an initial investment in net working capital of $240,000. The project is estimated to generate $1,920,000 in annual sales, with costs of $768,000. The tax rate is 32 percent and the required return on the project is 11 percent. Required: (a)What is the project's year 0 net cash flow (or cash flow from assets)? (b)What is the project's year 1 net cash flow (or cash flow from assets)? (c) What is the project's year 2 net cash flow (or cash flow from assets)?(d)What is the project's year 3 net cash flow (or cash flow from assets)? (e)What is the NPV? Joseph Ltd. has a stock of 4,000 containers valued at $. 5 each. During the year, the Company purchased 8,000 containers. It issued 80,000 containers to customers and received 74,000 containers from them; 80 containers were damaged of which 40 were repaired at a cost of $. 2 per container. The purchases are made at $. 10 per container but stocks are valued at $. 5 each to allow for depreciation. You are required to prepare the Containers Stock Account. An investment offers $10,000 per year for 15 years, with the first payment occurring one year from now. Assume the required return is 12 percent. a. What is the value of the investment today? (Do not round intermediate calculations and round your answer to 2 decimal places, e.g., 32.16.) b. What would the value be if the payments occurred for 40 years? (Do not round intermediate calculations and round your answer to 2 decimal places, e.g.. 32.16.) c. What would the value be if the payments occurred for 75 years? (Do not round intermediate calculations and round your answer to 2 decimal places, e.g.. 32.16.) d. What would the value be if the payments occurred forever? (Do not round intermediate calculations and round your answer to 2 decimal places, e.g., 32.16.) Curacao Pharmaceutical's cost of debt is 7 percent. The risk-free rate of interest is 3 percent. The expected return on the market portfolio is 8 percent. Effective tax rate is 25 percent. Its optimal capital structure is 60 percent debt and 40 percent equity. i. Calculate cost of equity, if Curacao's beta is estimated at 1.1. ii. Based on the answer in (b) (i), compute weighted average cost of capital (WACC). Summarize (half a page) "Rip Van Winkle" and explain whathistorical events Rip slept through. Then give and explain thetheme of the story. (The theme is the truth about life taught bythe literary w On May 1, 2021, Shamrock Construction Ltd. issued $700,000 of 20-year, 6% bonds at 100. The bonds pay interest semi-annually on November 1 and May 1. Shamrock has a calendar year end. (Credit account titles are automatically indented when the amount is entered. Do not indent manually. If no entry is required, select "No Entry" for the account titles and enter O for the amounts. Round answers to 0 decimal places, e.g. 5,276.) (a) Record the issuance of the bonds on May 1, 2021. (b) Record the first interest payment on November 1, 2021. (c) Prepare any adjusting entry required at December 31, 2021. (d) Record the second interest payment on May 1, 2022. (e) Assume that on May 1, 2022, immediately after paying the semi-annual interest, Shamrock redeemed 50% of the bonds at 98. Record the redemption of the bond, (f) Record the third interest payment on November 1, 2022, for the remaining bonds.